LSAT and Law School Admissions Forum

Get expert LSAT preparation and law school admissions advice from PowerScore Test Preparation.

 Administrator
PowerScore Staff
  • PowerScore Staff
  • Posts: 8919
  • Joined: Feb 02, 2011
|
#23625
Complete Question Explanation

Must Be True—PR. The correct answer choice is (D)

In this stimulus, Ellison's transgression is caught by illegal means, so the company cannot punish him. The correct answer choice will illustrate this same principle, barring punishment when the evidence represents "ill-gotten gain."

Answer choice (A): The principle at work in this question is that two wrongs don't make a right. Since this is not the principle illustrated in the stimulus, this answer choice is incorrect.

Answer choice (B): The principle in this choice is that hypocritical reporting of another's transgressions is not justified, which is not the same as the principle at work in the stimulus.

Answer choice (C): This answer choice may look tempting at first, but this principle would be that punishment cannot be based on the report of one who is guilty himself. This is slightly different from that illustrated in the passage, so this answer choice is incorrect.

Answer choice (D): This is the correct answer choice. In this scenario, like that presented in the stimulus, the evidence against the offender is gained illegally, precluding justified punishment in this case as well.

Answer choice (E): This choice illustrates the principle that those in power cannot justifiably punish one person for an act that went unpunished when committed by another.
 lanereuden
  • Posts: 147
  • Joined: May 30, 2019
|
#66168
This is actually law-related. Fruits from the poisonous tree!

Get the most out of your LSAT Prep Plus subscription.

Analyze and track your performance with our Testing and Analytics Package.